Difference between revisions of "Talk:2018 AMC 12B Problems/Problem 13"

(Put the discussion of the Shoelace Theorem on this page since I don't really want to complicate things up. I'd like to keep the solution page clean. Let me know if you disgree with this action.)
 
(No difference)

Latest revision as of 23:59, 24 October 2021

Remark (Coordinate Geometry)

Alternatively, use the Shoelace Theorem once you have found the coordinates in terms of $x$ and $y$. The $x$s and $y$s will conveniently cancel and the answer is (still) $\boxed{\textbf{C}}$.

~Fasolinka